0
$\begingroup$

Given a double slit experiment with one source of wavelength $\lambda$ shining through one slit and another source with wavelength $\lambda+\Delta\lambda$ shining through the other source, where $\Delta\lambda << \lambda$, how would the interference pattern behave? The slits are spaced a distance $2d$ apart and the center of the screen ($y=0$) is aligned with the centre of the two slits. (i.e. the positions of the slits are $y=d$ and $y=-d$) My guess is that it would move vertically with some velocity $v$. Any insight is appreciated, thanks!

$\endgroup$
1
  • 3
    $\begingroup$ This question is similar to: Can light waves cause beats?. If you believe it’s different, please edit the question, make it clear how it’s different and/or how the answers on that question are not helpful for your problem. $\endgroup$ Commented Jul 6 at 8:31

2 Answers 2

1
$\begingroup$

There exists a famous quote from Paul Dirac, saying that a photon does interfere only with itself. Hence, if you take two different lasers, they do not interfere. This is why the usual double slit experiment uses a single light source.

$\endgroup$
0
$\begingroup$

The fact that you have separate slits tied to each source will stop double slit interference. The source or sources need to "see" both slits.

Yes per Dirac every photon acts on its own! , but really it is better to say the EM field is unique for each photon. Even before the photon is emitted the excited electron in the source atom is already interacting with the EM field. Per Feynman integral the field in any apparatus will have modes, i.e. more resonant pathways, i.e more probable pathways. Your high school textbook explanation is almost 100 years old ... but its still taught today ... photons never interact or cancel each other.

$\endgroup$

Start asking to get answers

Find the answer to your question by asking.

Ask question

Explore related questions

See similar questions with these tags.